Jump to content

LangTu Mua Bui's Content

There have been 43 items by LangTu Mua Bui (Search limited from 08-06-2020)



Sort by                Order  

#599210 Đề thi chọn Đội tuyển Olympic SV ĐH Mỏ-Địa chất 2010-2011

Posted by LangTu Mua Bui on 20-11-2015 - 10:49 in Giải tích

Câu 1: Tính $\int_{-\dfrac{\Pi }{4}}^{\dfrac{\pi}{4}}ln(tanx+\sqrt{tan^2x+e^{sin^2x}})dx$

Câu 2:cho $f$ là một hàm có đạo hàm cấp $n$, liên tục trên $[a,b]$ và$f(x_1)=f(x_2)=...=f(x_n)$ và $a\leq x_1<x_2<...<x_n\leq b$

Ký hiệu: $M=\underset{x\epsilon [a,b]}{Max|f^{(n)}(x)|}$ Chứng minh $\forall x:|f(x)|\leq \dfrac{M}{n!}.\prod_{i=1}^{n}|x-x_i|$

Câu 3: Cho dãy
${U_n}$ có $U_0$ cho trước $0<U_0<1,U_{n+1}=\sqrt{\dfrac{1+U_n}{2}},V_n=\prod_{i=1}^{n}U_i$
Chứng minh dãy $V_n$ có giới hạn hữu hạn, tính giới hạn đó. Trong trường hợp $U_0\geq 1$ thì kết quả thế nào?

Câu 4: Tìm tất cả các hàm$f$ liên tục trên toàn trục số thỏa mãn:$f(x)-f(y)=\int_{x+2y}^{2x+y}f(t)dt,\forall x,y\epsilon R$

Câu 5: Cho$f$ là hàm có đạo hàm cấp 2 liên tục trên $[0,1]$ có $f(0)=f(1)=0$ và $\underset{x\epsilon [0,1]}{Min}f(x)=-1$. Chứng minh:
$\underset{x\epsilon [0,1]}{Max}f"(x)\geq 8.$

Câu 2 f(x_{1})=f(x_{2})=... có =0 vậy .
 




#714513 Chứng minh rằng các ma trận $M,N$ đồng dạng và các ma trận $P,...

Posted by LangTu Mua Bui on 18-08-2018 - 14:31 in Đại số tuyến tính, Hình học giải tích

Hai ma trận đồng dạng với nhau cho chúng có cùng đa thức cực tiểu như vậy ta có thể chéo hóa ma trận đó về dạng jordan .Đây là cách tổng quát cho mọi ma trận cho dù ma trận đó k có cơ sở để chéo hóa được.

 

 Các ma trận đường chéo $ A=diag(a_{ii}) $ có đặc điểm thuận tiện khi lũy thừa vì thế một đa thức P(x) thì  P(A) là một ma trận đường chéo có dạng $diag(P(a_{ii})) $ nó đúng cho cả các ma trận khối .
.
Ta  có tính chất nếu đa thức P(A)=0 thì nó là chia hết cho $g_{A} $ đa thức cực tiểu của ma trận A  
Xét P(N) là đa thức tối thiểu thì $diag(P(A+B) ,P(A-B) )$ Vì thế P(N)=0 khi chỉ khi P(A+B)=0 ,P(A-B) =0 tức P(N) là bội chung của của 2 đa thức cực tiểu A+B và A-B. .
Do  $g_{N} $là đa thức cực tiểu vì vậy là đa thức có bậc bé nhất suy ra nó là  bội chung nhỏ nhất của 2 đa thức cực tiểu $g_{A+B} $và $g_{A-B} $ 

Bây h ta sẽ đi chứng minh ma trận M cũng có đa thức cực tiểu là bội chung nhỏ nhất của $g_{A+B}$ và $g_{A-B}$
Xét thấy 

$\begin{pmatrix} A & B\\ B&A \end{pmatrix}=\frac{1}{2}\begin{pmatrix} (A+B)+(A-B) & (A+B)-(A-B) \\ (A+B)-(A-B)& (A+B)+(A-B) \end{pmatrix}$

$\begin{pmatrix} A &B \\ B&A \end{pmatrix}^{2}=\frac{1}{2}\begin{pmatrix} (A+B)^{2}+(A-B)^{2} & (A+B)^{2}-(A-B)^{2} \\ (A+B)^{2}-(A-B)^{2}& (A+B)^{2}+(A-B)^{2} \end{pmatrix}$
.. ..
$P(M)=\frac{1}{2}\begin{pmatrix} P(A+B)+P(A-B) & P(A+B)-P(A-B)\\ P(A+B)-P(A-B)& P(A+B)+P(A-B) \end{pmatrix}$
 

Tương tự nếu $P(M)=0 $thì $P(A+B)+P(A-B)=0$ và $P(A+B)-P(A-B)=0$ điều đó có nghĩa là P(M)=0 khi và chỉ khi$ P(A+B) $và $P(A-B) $đều bằng 0.
Dẫn tới các đa thức thỏa mãn$ g(M)=0$ thì chúng là bội chung của các đa thức cực tiểu $(A+B) $và $(A-B) $

Nếu là đa thức cực tiểu số bậc nhỏ nhất thì nó là duy nhất và là bội chung nhỏ nhất của 2 đa thức cực tiểu của 2 ma trận $(A+B) $và $(A-B) .$

Vậy ta đã chứng minh 2 ma trận M và N cùng đa thức cực tiểu vì thế chúng đồng dạng với nhau.




#714547 Chứng minh rằng các ma trận $M,N$ đồng dạng và các ma trận $P,...

Posted by LangTu Mua Bui on 19-08-2018 - 10:40 in Đại số tuyến tính, Hình học giải tích

Hai ma trận có cùng đa thức tối tiểu không nhất thiết phải đồng dạng với nhau, điều ấy chỉ đúng với ma trận cỡ $3\times 3$ và sai với mọi $n>3$.

 

http://www.mathcount...ic-polynomials/

Để sửa và bổ sung thêm đoạn chứng minh cùng số khối 
Hai ma trận vuông cùng cấp đồng dạng với nhau khi và chỉ khi chúng cùng dạng chuẩn tắc 
Lời bổ sung sẽ thêm sau đây




#714549 Chứng minh rằng các ma trận $M,N$ đồng dạng và các ma trận $P,...

Posted by LangTu Mua Bui on 19-08-2018 - 11:22 in Đại số tuyến tính, Hình học giải tích

Hai ma trận đồng dạng với nhau khi chúng cùng dạng chuẩn tắc như vậy ta có thể chéo hóa ma trận đó về dạng jordan .Đây là cách tổng quát cho mọi ma trận cho dù ma trận đó k có cơ sở để chéo hóa được.

Đầu tiên đi chứng minh chúng cùng đa thức đặc trưng :

 

 Các ma trận đường chéo $ A=diag(a_{ii}) $ có đặc điểm thuận tiện khi lũy thừa vì thế một đa thức P(x) thì  P(A) là một ma trận đường chéo có dạng $diag(P(a_{ii})) $ nó đúng cho cả các ma trận khối .
.
Ta  có tính chất nếu đa thức P(A)=0 thì nó là chia hết cho $g_{A} $ đa thức cực tiểu của ma trận A  
Xét P(N) là đa thức tối thiểu thì $diag(P(A+B) ,P(A-B) )$ Vì thế P(N)=0 khi chỉ khi P(A+B)=0 ,P(A-B) =0 tức P(N) là bội chung của của 2 đa thức cực tiểu A+B và A-B. .
Do  $g_{N} $là đa thức cực tiểu vì vậy là đa thức có bậc bé nhất suy ra nó là  bội chung nhỏ nhất của 2 đa thức cực tiểu $g_{A+B} $và $g_{A-B} $ 

Bây h ta sẽ đi chứng minh ma trận M cũng có đa thức cực tiểu là bội chung nhỏ nhất của $g_{A+B}$ và $g_{A-B}$
Xét thấy 

$\begin{pmatrix} A & B\\ B&A \end{pmatrix}=\frac{1}{2}\begin{pmatrix} (A+B)+(A-B) & (A+B)-(A-B) \\ (A+B)-(A-B)& (A+B)+(A-B) \end{pmatrix}$

$\begin{pmatrix} A &B \\ B&A \end{pmatrix}^{2}=\frac{1}{2}\begin{pmatrix} (A+B)^{2}+(A-B)^{2} & (A+B)^{2}-(A-B)^{2} \\ (A+B)^{2}-(A-B)^{2}& (A+B)^{2}+(A-B)^{2} \end{pmatrix}$
.. ..
$P(M)=\frac{1}{2}\begin{pmatrix} P(A+B)+P(A-B) & P(A+B)-P(A-B)\\ P(A+B)-P(A-B)& P(A+B)+P(A-B) \end{pmatrix}$
 

Tương tự nếu $P(M)=0 $thì $P(A+B)+P(A-B)=0$ và $P(A+B)-P(A-B)=0$ điều đó có nghĩa là P(M)=0 khi và chỉ khi$ P(A+B) $và $P(A-B) $đều bằng 0.
Dẫn tới các đa thức thỏa mãn$ g(M)=0$ thì chúng là bội chung của các đa thức cực tiểu $(A+B) $và $(A-B) $

Nếu là đa thức cực tiểu số bậc nhỏ nhất thì nó là duy nhất và là bội chung nhỏ nhất của 2 đa thức cực tiểu của 2 ma trận $(A+B) $và $(A-B) .$

Tiếp theo chứng minh chúng cùng mỗi khối liên kết tương ứng bằng nhau 

Tức thỏa mãn định lý số chiều gọi S_{ik} là số khối của đa thức liên kết 
Gọi là g là đa thức tối thiểu của ma trận M $g=g^{p_{1}}_{1}...g^{p_{r}}_{n} $
$S_{ik}=\frac{rank g^{k-1}_{i} -2rank g^{k}_{i}+rank g^{k+1}_{i} }{deg g_{i}} $
 

Với ma trận N ta có $rankg_{i}(A)=\begin{pmatrix} g_{i}(A+B) &0 \\ 0 & g_{i}(A-B) \end{pmatrix}$ suy ra $rank g_{i}N)=rank(g_{i}(A+B))+rank g_{i}(A-B) $ 
 

Với ma trận M nhờ cộng theo hàng ta được $ rank g_{i}(M)$=rank $\begin{pmatrix} g_{i}(A+B)+g_{i}(A-B) & g_{i}(A+B)-g_{i}(A-B)\\ g_{i}(A+B)-g_{i}(A-B) & g_{i}(A+B)+g_{i}(A-B) \end{pmatrix} =$rank $\begin{pmatrix} g_{i}{A+B} &g_{i}{A+B} \\ -g_{i}{A-B} &g_{i}(A-B) \end{pmatrix}$
Ta thấy mỗi hạng của ma trận của khối thứ 1 khi biến đổi sơ cấp với các hạng của khối thứ 2 đều không thể bằng 0 điều này chứng tỏ  hạng của ma trận M bằng hạng $rank g_{i}(A+B) +rank g_{i}(A-B) $ 
Điều này chứng tỏ số khối liên kết của 2 ma trận tương tứng bằng nhau .

Vậy ta đã chứng minh 2 ma trận M và N cùng đa thức cực tiểu vì thế chúng đồng dạng với nhau.




#598963 $\int_{0}^{T}f(x)dx=\int_{a}^...

Posted by LangTu Mua Bui on 18-11-2015 - 17:24 in Giải tích

Làm như mấy bạn là sai rùi tích phân không phụ thuộc vào biến nhưng đặt xong cận nó lại thay đổi 

$g(x)=\int_{x}^{x+T}f(t)dt $
 
$\Rightarrow g'(x)=f(x+T)-f(x) =0 \forall x \in R \Rightarrow g(x)=$const 



#713828 $trace(ABAB) \le trace(A^2B^2)$

Posted by LangTu Mua Bui on 04-08-2018 - 21:39 in Đại số tuyến tính, Hình học giải tích

Thực ra ma trận cũng là 1 dạng hệ thống các thành phân với quy ước nhân công được định nghĩa rõ rạng .

Nếu đọc liên quan đến Tensor sẽ rõ nó nhân tính toán với hệ thống sẽ rõ .

Nên hiểu dưới dạng với mỗi thành phần của ma trận  $a_{ij}e_{i}e_{j}$ với $ e_{i}e_{j}$ là cơ sở điat 

Tích vô hướng trên của 2 số hạng =1 khi có cùng cơ sở điat và =0 nếu có 1 chỉ số khác 0 .

Thì Tích vô hướng giữa 2 ma trận $ \left \langle A; B  \right \rangle =AB^{T}=tr(AB) $ 

Ta có $ \left \langle A;BC \right \rangle=\left \langle C^{T}A;B \right \rangle  $
Mặt khác do A và B là các ma trận đối xứng 

Từ đó suy ra  $  tr(ABAB-A^{2}B^{2})=tr(A(AB-BA)B) =\left \langle  AB-BA;AB   \right \rangle \geqslant 0   $

Đặt ma trận C=AB suy ra TÍch vô hướng trên tương đương với$ \sum_{i,j}^{n} c^{2}_{ij}-c_{ij}c_{ji} \geq  0$ 

Sử dụng bất đẳng thức Cauchy quen thuộc cho$  \sum_{i,j}^{n}  c^{2}_{ij}  \geq \sum_{ij}^{n}c_{ij}c_{ji} $(Đpcm) .




#714054 Hạng của ánh xạ tuyến tính

Posted by LangTu Mua Bui on 08-08-2018 - 19:39 in Đại số tuyến tính, Hình học giải tích

Sử dụng định lý : $n=dim(imf)+dim(kerf) $ Với  $dim (imf)=rank(f)= rank (A) $với A là ma trận biểu diễn ánh xạ tuyến tính f.
Vì thế ta luôn có không gian ảnh có số chiều nhỏ hơn không gian nguồn ( n là số chiều không gian nguồn ở định lý trên ) 
Từ đây ta cần coi $rank(g) $là tập nguồn thì $rank(f.g) $là số chiều của kg tập ảnh 




#601477 Đề thi chọn đội tuyển Olympic toán sinh viên 2013 học viện tài chính...

Posted by LangTu Mua Bui on 03-12-2015 - 20:56 in Thảo luận về các kì thi, các kì kiểm tra Toán sinh viên

Câu 5 
$\lim_{x\to\infty}g(x)=\lim_{x\to\infty}\frac{e^{x}g(x)}{e^{x}}(Lopitan)$

$ =\lim_{x\to\infty}\frac{e^{x}(g(x)+g'(x))}{e^{x}} \Rightarrow \lim_{x\to\infty}g'(x)=0 $

$\Leftrightarrow 0=\lim_{x\to\infty}f(g(x)) $ Do f và g là 2 hàm liên tục nên $\lim_{x\to\infty}f(g(x)) $

$=f(\lim_{x\to\infty}g(x))=f(c) \Rightarrow f(c)=0$



#601731 Đề thi OLP toán sinh viên cấp trường đh Kinh tế quốc dân 2013

Posted by LangTu Mua Bui on 05-12-2015 - 16:00 in Thảo luận về các kì thi, các kì kiểm tra Toán sinh viên

Câu 6
 
Dễ dàng Cm được pt $f(x)=x $có nghiệm trên$ (a;b) $

Vì vậy tồn tại các đoạn sao cho $f(x)<0 \forall x \in (a;b)$ và  $f(x)>0 \forall x \in (c;d)$ 

Xét trên đoạn $f(x)<0 \forall x \in (a;b)$ Luôn tồn tại$ \alpha >0$và x lập thành CSC trên $(a;b)$
 
Chọn $x_{1}$ thỏa mãn :$x_{1}>a; x_{1}+n\alpha<b.$
  
Ta có $(n+1)\left ( x+\frac{n\alpha}{2} \right )=\sum_{k=0}^{n}(x+k\alpha)$

$ \Leftrightarrow g(x_{1})= \sum_{k=0}^{n}\left ( f(x+k\alpha)-(x+k\alpha) \right )
\Leftrightarrow  \sum_{k=0}^{n}\left ( f(x_{1}+k\alpha)-(x_{1}+k\alpha) \right ) <0$

Tương tự trên  $\forall x\in (c;d) ;f(x)>0$ Chọn $x_{2}$ thỏa mãn $x_{2}>c ; x_{2}+n\alpha<d $
 
$\Rightarrow   g(x_{2})=\sum_{k=0}^{n}\left ( f(x_{2}+k\alpha)-(x_{2}+k\alpha) \right )>0 $

 
$\Rightarrow g(x_{1}).g(x_{2})<0 \exists c \in (x_{1};x_{2}) $sao cho $g(c)=0$ ĐPCM
 




#716993 $AB$ và $BA$ có cùng đa thức biểu diễn

Posted by LangTu Mua Bui on 28-10-2018 - 19:29 in Đại số tuyến tính, Hình học giải tích

Với $A_{i},B_{j}$ là các khối được tao từ các cột và các hàng của A và B.
Áp dụng vào bài toán ta được kết quả 
$C=\begin{bmatrix} B &I \end{bmatrix}.\begin{bmatrix} A\\ kI\\ \end{bmatrix} ;C^{T}=\begin{bmatrix} A &kI \end{bmatrix}.\begin{bmatrix} B\\ I\\ \end{bmatrix}$
Do $\det(C)=\det(C^{T})$ nên $\det(AB+kI)=\det(BA+kI)$ ta có kết quả đa thức đặc trưng của 2 ma trận AB và BA là bằng nhau.



#601536 Đề thi Olympic sinh viên năm 2013 trường ĐHKHTN - ĐHQG Hà Nội

Posted by LangTu Mua Bui on 04-12-2015 - 09:14 in Thảo luận về các kì thi, các kì kiểm tra Toán sinh viên

Tối nay ngồi post lời giải vậy :D

Giả sử $g(b) \neq 0$, không mất tính tổng quát, giả sử $g(b)>0$, khi đó

$$\lim_{x \to +\infty} \varphi' (x)=g(b) $$

$$\Leftrightarrow \forall \epsilon>0, \exists x_0, \forall x \ge x_0 \rightarrow |\varphi'(x)-g(b)|<\epsilon $$

$$\Rightarrow \varphi'(x)>g(b)-\epsilon$$

Chọn $\epsilon$ đủ nhỏ sao cho $g(b)-\epsilon>0$, suy ra $\int_{x_0}^x \varphi' (t)dt>\int_{x_0}^x (g(b)-\epsilon)dt $

$$\Leftrightarrow \varphi(x)>(g(b)-\epsilon)(x-x_0)+\varphi(x_0)$$

$$\Rightarrow \lim_{x \to +\infty} \varphi(x)=+\infty$$

Mâu thuẫn.

Vậy $g(b)=0$

g(b)=0,0000...1 thì sao 




#601542 Đề thi Olympic sinh viên năm 2013 trường ĐHKHTN - ĐHQG Hà Nội

Posted by LangTu Mua Bui on 04-12-2015 - 09:31 in Thảo luận về các kì thi, các kì kiểm tra Toán sinh viên

Bài 4. Cho $f:\left( {0, + \infty } \right) \to $ thỏa mãn các điều kiện sau
i. $\mathop {\lim }\limits_{x \to + \infty } \left( {f\left( {x + 1} \right) - f(x)} \right) = + \infty $.
ii. $f$ bị chặn trên mọi khoảng con hữu hạn của $\left( {0, + \infty } \right)$.
Chứng minh rằng $\mathop {\lim }\limits_{x \to + \infty } \frac{{f(x)}}{x} = + \infty $.

Không rảnh nhưng thấy bài toán hay nên post lời giải lên… :lol:

Giải.

Theo giả thiết $f$ bị chặn trên $\left( {0, + \infty } \right)$. Lại có $\mathop {\lim }\limits_{x \to + \infty } \left( {f\left( {x + 1} \right) - f(x)} \right) = + \infty $ nên với mọi $M > 0,\exists {x_0} > 0$ sao cho với mọi $x \ge {x_0}$ ta có $f\left( {x + 1} \right) - f(x) > M$.
Sử dụng liên tiếp bất đẳng thức trên ta suy ra

$M + f(x) < f\left( {x + 1} \right) < f\left( {x + 2} \right) - M < ... < f\left( {x + n} \right) - \left( {n - 1} \right)M$.

Suy ra

$f\left( {x + n} \right) > nM + f(x)$ với mọi $x \ge {x_0}$,

nói riêng ta có $f\left( {{x_0} + n} \right) > nM + f({x_0}) \Rightarrow \frac{{f\left( {{x_0} + n} \right)}}{n} > M + \frac{{f({x_0})}}{n}$.
Do $f$ bị chặn nên với $n$ đủ lớn suy ra $\frac{{f\left( {{x_0} + n} \right)}}{n} > M$ tức $f\left( {{x_0} + n} \right) > 0$. Khi đó với $n$ đủ lớn luôn tồn tại ${x_0} \le n$. Do đó

$\frac{{f\left( {{x_0} + n} \right)}}{{{x_0} + n}} \ge \frac{{f\left( {{x_0} + n} \right)}}{{2n}} > \frac{M}{2} + \frac{{f({x_0})}}{{2n}}$.

Suy ra với mọi $x > {x_0} + 2n$ thì $\frac{{f(x)}}{x} > \frac{M}{2}$ với mọi $M > 0$. Do đó $\mathop {\lim }\limits_{x \to + \infty } \frac{{f(x)}}{x} = + \infty $.
Bài toán được chứng minh.

 

Bài này sao không sử dụng ĐL stolez luôn với $a_{n}=f(n)$ 




#601708 Đề thi Olympic toán sinh viên ĐH BK HN 2013

Posted by LangTu Mua Bui on 05-12-2015 - 11:58 in Thảo luận về các kì thi, các kì kiểm tra Toán sinh viên

Câu 5 
$\left\{ \begin{array}{l} f'\left( x \right) > 0\\ f\left( {f'\left( x \right)} \right) = - f(x) \end{array} \right.$

Ta có $f(f'(x))=-f(x)$

Thay $ x=f'(x) \Rightarrow f(f'(f'(x))))=f(x)$ Do $f'(x)>0 \Rightarrow f'(f'(x))=x  (1)$

$ \Rightarrow  f(f'(x))=-f(x)$

Do hàm khả vi cấp 2 nên Ta  đạo hàm 2 vế theo x $ \Rightarrow f''(x)f'(f'(x))=-f'(x)(2)$
 
Từ $ (1)(2)f''(x).f'(f'(x))\Leftrightarrow f''(x)x+f'(x)=0$

$ \Rightarrow f(x)=a\ln{x}+b$ Thay ngược lại đề bài $\Rightarrow  a=b=1 \Rightarrow f(x)=\ln{x} $



#601709 Đề thi Olympic toán sinh viên ĐH BK HN 2013

Posted by LangTu Mua Bui on 05-12-2015 - 11:59 in Thảo luận về các kì thi, các kì kiểm tra Toán sinh viên

 Cách khác cho câu 3

$u(x)\leq 1+\int_{0}^{x}\frac{\varphi '(t)u(t)dt}{\varphi (t)}$

Dễ thấy $ u(0) \leq 1$
 
$\Leftrightarrow u(x)-\varphi (x)  \leq  \int_{0}^{x} \frac{\varphi '(t)u(t)dt}{\varphi (t)}-\int_{0}^{x}\varphi '(t)dt=\int_{0}^{x}\left (\varphi '(t)( \frac{u(t)-\varphi (t)}{\varphi (t)})  \right ) dt $

Do $\varphi (t)$ đồng biến và $\varphi(0)=1 \Rightarrow \varphi (t)\geq 1 \forall t\in [0;\infty) $

$\Rightarrow u(x)-\varphi (x) \leq  \int_{0}^{x}\left (\varphi '(t)(u(x)-1)  \right ) dt=u(x)-\varphi (x)-\int_{0}^{x}\varphi '(x) $

$\Rightarrow \varphi(x)<1-\int_{0}^{x}u'(t)\varphi (t)dt ;\varphi(x)\geq 1 \forall x\in [0;\infty] \Rightarrow  u'(t)<0 $

Xét hàm số $g(x)=u(x)-\varphi (x) $

$g'(x)=u'(x)-\varphi' (x) <0 ;g(0)=u(0)-\varphi (0)<0 \Rightarrow g(x)<0 \forall x\in [0;\infty]$



#601684 Đề thi Olympic toán sinh viên 2013 ĐHSP HCM môn giải tích

Posted by LangTu Mua Bui on 05-12-2015 - 01:33 in Thảo luận về các kì thi, các kì kiểm tra Toán sinh viên

Câu 4 Ta sử dụng Larange để giải bài này

Xét $g(x)=\ln{\sin{f(x)}}$


Theo đl Larange ta có 
$g'(c)=\frac{g(b)-g(a)}{b-a}=\frac{\ln{\sin{f(b)}}-\ln{\sin{f(a)}}}{b-a}$với $g'(c)=f'(c)\cot{f(c)}$(1)

$\frac{1}{a-c}<0<\frac{1}{b-a}<\frac{1}{b-c}$(2) 

Mặt khác $\ln{\sin{f(b)}}-\ln{\sin{f(a)}}<2$ (3)

Từ (1)(2)(3) Ta được đpcm



#601682 Đề thi Olympic toán học sinh viên 2012 Đại Học BK Hà Nội

Posted by LangTu Mua Bui on 05-12-2015 - 01:10 in Thảo luận về các kì thi, các kì kiểm tra Toán sinh viên

Câu 1
$x_n=\underbrace{\sqrt[3]{6+\sqrt[3]{6+...+\sqrt[3]{6}}}} $

Ta có $2=\sqrt[3]{8}=\sqrt[3]{6+2}=\sqrt[3]{6+\sqrt[3]{8}}=\sqrt[3]{6+\sqrt[3]{6+\sqrt[3]{6+...}}}$

$\Rightarrow \lim_{n\to\infty}6^n(2-x^{n})=0$



#601683 Đề thi Olympic toán học sinh viên 2012 Đại Học BK Hà Nội

Posted by LangTu Mua Bui on 05-12-2015 - 01:21 in Thảo luận về các kì thi, các kì kiểm tra Toán sinh viên

Xét $g(x)=\int_{0}^{x}f(t)dt$ Do f(x) khả vi 2 lần nên g(x) khả vi 3 lần 

Ta có$ g'(x)=f(x) ..$
 
Khai triển maclaurank $\Rightarrow g(x)=g(0)+g(0)x+\frac{g'(0)x^{2}}{3}+\frac{g''(0+a(x-x_{0}))x^{3}}{6}$ (0<a<1)

Thay x=1 ta được $g(1)=\int_{0}^{1}=f(0)+\frac{f'(0)}{2}+\frac{f''(\theta (x))}{6}$ với $c=0<\theta (x)<1$ Ta đpcm 
 



#605905 Ôn thi Olympic Toán học sinh viên 2015 [Giải tích]

Posted by LangTu Mua Bui on 29-12-2015 - 12:14 in Thảo luận về các kì thi, các kì kiểm tra Toán sinh viên

Bài 19: (thật ra tương tự câu 14)

Cho $f:\mathbb{R} \to \mathbb{R}$ liên tục sao cho $\forall x \in \mathbb{R} , \int_0^1 f(xt)dt=0$

Chứng minh $$f(x)=0 \;\;,\forall x \in \mathbb{R}$$

Với $x=0 \Rightarrow f(0)=0$

 Với $x \neq 0$ Đặt$u=xt \Rightarrow g(x)=\frac{\int_{0}^{x}f(u)du}{x}=0$

$\Rightarrow \int_{0}^{x}f(u)du=0\forall u $

Do f(x) liên tục nên hàm g(x) khả vị nên Đạo hàm 2 vê ta được $ \Rightarrow g'(x)=f(x)=0
f(0)=0$

Hàm f(x) là hàm hằng $$\Rightarrow f(x)=0$$